Mathe-Marathon Uni - Seite 3

Neue Frage »

tmo Auf diesen Beitrag antworten »

Zitat:
Original von Airblader
Ohne überhaupt darüber nachzudenken habe ich einfach das Gefühl, dass es aus offensichtlichen Gründen d=42 sein muss... Big Laugh

air


Jetzt wo die Lösung schon da steht, kann man ja mit weiteren Zahlen, die die Eigenschaft erfüllen, rausrücken:

Der kleine Satz von Fermat besagt doch gerade, dass die Eigenschaft erfüllt. Den sollte man aber auch als Nicht-Algebraiker kennen Teufel
IfindU Auf diesen Beitrag antworten »

Also d = 2 kann man leicht einsehen, und mit tmo sind wir schon bei d = 86. Jetzt fehlt nur noch der Faktor 21 zu HALs Lösung Augenzwinkern
Airblader Auf diesen Beitrag antworten »

@ tmo

Ja, an den kleinen Fermat dachte ich fünf Sekunden danach auch -- aber meine Antwort war ja auch mehr Spaß als Ernst. Augenzwinkern

air
Reksilat Auf diesen Beitrag antworten »

Es sei ein Primteiler von .
Es ist und insofern ist quadratfrei.

Sei teilerfremd zu , dann hat in der multiplikativen Gruppe die Ordnung . Nun ist nach Voraussetzung und dies ist äquivalent zu .
Dies gilt genau für die Primzahlen 2, 3, 7 und 43.

Es ergibt sich

Zitat:
Aufgabe 22:
(Aus einer Folge von Futurama)
Mit einer Körpertauschmaschine können zwei Personen sich in den Körper des jeweils anderen transferieren. Leider kann diese Prozedur für jedes Paar nur höchstens einmal durchgeführt werden. Nun haben Personen wild durcheinander ihre Körper vertauscht und keiner weiß mehr, welche Vertauschungen genau durchgeführt wurden. Wie viele zusätzliche Personen benötigt man nun, um jeden wieder in seinen richtigen Körper zurück transferieren zu können?


Gruß,
Reksilat.
tmo Auf diesen Beitrag antworten »

Zitat:
Original von Reksilat
Sei teilerfremd zu , dann hat in der multiplikativen Gruppe die Ordnung .


Zumindest gibt es solche x. Für jedes teilerfremde x gilt dies natürlich nicht.


Zur neuen Aufgabe habe ich ein paar Fragen (noch keine Ahnung, ob es relevant ist, aber kann man ja mal klären):

Sollen die zusätzlichen Personen "Bauernopfer" sein oder sollen die nach der ganzen Aktion auch wieder in ihrem eigenen Körper sein?

Wie ist "für jedes Paar nur einmal gemeint?" Merkt sich die Maschine die Körper oder die Seelen? (ich nehme an die Seelen)

Mit "keiner weiß mehr" ist gemeint, dass die n Personen untereinander gar nicht mehr tauschen, weil sie nicht wissen, ob es noch geht?
Reksilat Auf diesen Beitrag antworten »

Ja:
- die zusätzlichen Personen sollen auch wieder in ihren Körper zurückkommen.
- die Maschine merkt sich die Körper
- bei einer weiteren Vertauschung dürfen nicht mehr beide zum Kreis der n Personen gehören.

Es geht letztlich darum, ein m>n zu finden, so dass sich jedes Element der als Produkt von paarweise verschiedenen Transpositionen aus darstellen lässt.

Gruß,
Reksilat.
 
 
Airblader Auf diesen Beitrag antworten »

Amy: "Oh no! Is it possible to get everyone back to normal using four or more bodies?"
Dr. Farnsworth: "I'm not sure. I'm afraid we need to use ..."

Eine von vielen hervorragenden Folgen! Freude
Ich könnte jetzt einfach das 11-seitige Paper zitieren, das der Drehbuchautor zu diesem Thema veröffentlicht hat (der vorher übrigens seinen PhD in Mathematik an der Harvard University gemacht hat), aber das wäre Betrug. Augenzwinkern

air
Reksilat Auf diesen Beitrag antworten »

Ich habe das Paper jetzt auf die Schnelle nicht gefunden, bin mir aber sicher, dass man den Beweis auch auf eine halbe Seite packen kann. Augenzwinkern

Ich kann's aber auch auflösen. Ups

Gruß,
Reksilat.
Airblader Auf diesen Beitrag antworten »

Sicher geht das auch auf eine halbe Seite, das Paper macht eben ein bisschen mehr. Ich beabsichtige übrigens nicht, aufzulösen -- ich hätte auch gar keine nächste Frage. Big Laugh

Aber irgendjemand bekommt das doch wohl hin? smile

air
Reksilat Auf diesen Beitrag antworten »

Da es jetzt schon eine Woche her ist, löse ich mal auf:


Zitat:
Lösung zu Aufgabe 22:

Ein Element aus lässt sich als Produkt elementfremder Zykeln schreiben.
Sei solch ein Zykel für
Dann nehmen wir uns zwei Elemente und rechnen nach:

(Wir lesen die Permutationen von links nach rechts)

Ist nun ein Produkt disjunkter Zykeln, so ist:

Die ganzen können wir, wie oben gesehen, als Produkt von Transpositionen aus schreiben und keine Transposition wiederholt sich. Am Ende kommt dann bei einer ungeraden Anzahl von Zykeln noch ein dazu.

Es genügen also zwei weitere Personen, um alle Körper wieder zurückzutauschen.



Ein neues Rätsel habe ich aber nicht. Zunge

Gruß,
Reksilat.
Iorek Auf diesen Beitrag antworten »

Zitat:
Original von Reksilat
Ein neues Rätsel habe ich aber nicht. Zunge


Dann erlaube ich mir einfach, für dich einzuspringen und etwas leichte Analysis in den Thread zu bringen. Augenzwinkern

Zitat:
Aufgabe 23:
Es seien mit sowie monoton mit .

Man zeige: die Funktion ist stetig genau dann, wenn sie surjektiv ist.
Alive-and-well Auf diesen Beitrag antworten »

Fall 1: ""
Sei Stetig.

Da der ZWS gilt nimmt jeden Wert zwischen und an!
Also: . muss also Surjektiv sein!

Fall 2: ""
Sei Surjektiv.

Annahme: ist NICHT stetig! ( besitzt also eine Sprungstelle)
Es gibt also einen Punkt q mit:


Betrachtet man das Intervall so folgt aus der Monotonie, das die Zahlen innerhalb des Intervalls nur von der Zahl q abgebildet werden können. Da f(q) allerdings nicht existiert kann h nicht Surjektiv sein!
Widerspruch zur Annahme!

ist stetig

w.z.B.w
tmo Auf diesen Beitrag antworten »

Zitat:
Original von Alive-and-well
Annahme: ist NICHT stetig! ( besitzt also eine Sprungstelle)
Es gibt also einen Punkt q mit:



Hier wäre noch zu zeigen, dass eine Sprungstelle wirklich die einzig mögliche Unstetigkeit wäre, d.h. es ist zu zeigen, dass die einseitigen Grenzwerte existieren. Da geht die Monotonie noch entscheidend ein.


Zitat:
Original von Alive-and-well
Betrachtet man das Intervall so folgt aus der Monotonie, das die Zahlen innerhalb des Intervalls nur von der Zahl q abgebildet werden können. Da f(q) allerdings nicht existiert kann h nicht Surjektiv sein!
Widerspruch zur Annahme!


Das sollte man noch etwas besser formulieren, so macht dies noch nicht richtig Sinn.


Einen direkter Beweis mit dem Epsilon-Delta-Kriterium wäre m.E. etwas schneller. Durch den Übergang zu sieht man übrigens, dass nur die Rechtsstetigkeit zu zeigen ist. Damit spart man sich etwas technischen Aufwand.

@Alive-and-well: Willst du deine Ausführungen noch etwas verbessern und dann mit der nächsten Aufgabe weitermachen?


PS: Die Aufgabe ist insofern interessant, als dass sie eine Gefahr birgt. Nicht selten hab ich gesehen (z.B. hier im Board), dass die Stetigkeit der Exponentialfunktion mit Hilfe der Rückrichtung (Konkret wurde der Logarithmus benutzt, um die Stetigkeit zu zeigen) nachgewiesen wird. Dabei beißt sich die Katze aber meist selbst in den Schwanz, weil die Surjektivität der Exponentialfunktion (Und damit die Existenz des Logarithmus) doch meist eine Folgerung aus der Stetigkeit ist.


edit: Sorry @ Iorek, jetzt bist du ja auch online. Warst du eben nocht nicht, oder? verwirrt
Alive-and-well Auf diesen Beitrag antworten »

Zitat:
Original von tmo
Hier wäre noch zu zeigen, dass eine Sprungstelle wirklich die einzig mögliche Unstetigkeit wäre, d.h. es ist zu zeigen, dass die einseitigen Grenzwerte existieren. Da geht die Monotonie noch entscheidend ein.

Keine Ahnung ob das so geht aber:
Sei:




Dann sollte gelten: (f monoton wachsend)




So oder so ähnlich dann auch für rechts !


Zitat:
Original von tmo
Das sollte man noch etwas besser formulieren, so macht dies noch nicht richtig Sinn.



Es gilt )
Und ebenso )



Dieses Intervall enthält unendlich viele (reele) Zahlen, da f(q) allerdings nur eine dieser Zahlen abbilden kann, folgt das die Funktion NICHT Surjektiv sein kann.
tmo Auf diesen Beitrag antworten »

Die Abschätzung kriegt man nur kleinergleich würde ich sagen:

Aber das ist schon ok nun. Nächste Aufgabe bitte Freude
Alive-and-well Auf diesen Beitrag antworten »
RE: Mathe-Marathon Uni
Zitat:
Aufgabe 24

Bereich:Analysis

Entwickeln Sie die folgende Funktione in eine Taylorreihe um den Punkt . Diskutieren Sie dabei ausführlichst das Verhalten des Restgliedes für und geben Sie schließlich Reihe und Gültigkeitsbereich derselben an.





Taylor war meine ich noch nicht :P
tmo Auf diesen Beitrag antworten »

Wir definieren mal und bemerken:




Damit sieht man:

, also



Insbesondere erhält man



, also ergibt sich:




Mehr oder weniger offensichtlich verschwindet das Restglied für alle x.

Damit erhalten wir

, also




Dasselbe Resultat erhält man, wenn man einfach schreibt und dann die Exponentialreihe einsetzt.
Iorek Auf diesen Beitrag antworten »

Zitat:
Original von tmo
Sorry @ Iorek, jetzt bist du ja auch online. Warst du eben nocht nicht, oder? verwirrt


Das passt schon so, ich hatte bis grade höllisch viel zu tun, hatte nur mal kurz reingeschaut und deine passende Antwort schon gesehen. Augenzwinkern

Interessant fand ich die Aufgabe damals auch, nur dass sie als Klausuraufgabe gestellt war, fand ich nicht ganz so schön...hab mich bei der nicht trivialen Richtung ganz schön verrannt und Punkte liegen lassen.
tmo Auf diesen Beitrag antworten »

Zitat:
Original von Iorek
Interessant fand ich die Aufgabe damals auch, nur dass sie als Klausuraufgabe gestellt war, fand ich nicht ganz so schön...hab mich bei der nicht trivialen Richtung ganz schön verrannt und Punkte liegen lassen.


Ja, in einer Klausur kann die Aufgabe unangenehm werden, da die Rückrichtung (Wenn man denn mal den Beweis im Kopf geführt hat und aufschreiben will) doch vielen technischen Details unterliegt, auf die dann gerade in einer Ana I-Klausur (war doch bestimmt Ana I oder?) doch noch sehr penibel geachtet wird.

Ich würde die Rückrichtung so angehen:

Wie bereits gesagt, reicht es die Rechtstetigkeit zu zeigen (Zack, der erste Punktabzug...). Sei also und . Ist , so ist nichts zu zeigen (nächster Punktabzug...). Andernfalls wähle mit und setze (Spätestens jetzt wäre ich durchgefallen Big Laugh )
Iorek Auf diesen Beitrag antworten »

Zitat:
Original von tmo
auf die dann gerade in einer Ana I-Klausur (war doch bestimmt Ana I oder?) doch noch sehr penibel geachtet wird.

Ich würde die Rückrichtung so angehen:

Wie bereits gesagt, reicht es die Rechtstetigkeit zu zeigen (Zack, der erste Punktabzug...). Sei also und . Ist , so ist nichts zu zeigen (nächster Punktabzug...). Andernfalls wähle mit und setze (Spätestens jetzt wäre ich durchgefallen Big Laugh )


Ja, das war Ana 1, und ja, für jeden Kleinscheiß wurden Punkte abgezogen (Beispiel aus den Übungen: gibt 0 Punkte, da dieser Grenzwert noch nicht bekannt ist, die Umformung und ein Verweis auf die Anwendung der Grenzwertsätze auf die (bekannten) konvergenten Folgen wäre nötig gewesen). Für deine Argumentation hätte es also höchstens Minuspunkte gegeben. Big Laugh
Monoid Auf diesen Beitrag antworten »

Wann kommt denn die nächste Aufgabe?

Mmm
tmo Auf diesen Beitrag antworten »

Ich hatte noch auf eine Bestätigung gewartet, aber jetzt mache ich dann doch mal weiter, damit es nicht zum Erliegen kommt.

Aufgabe 25

Man zeige die Ungleichung vom arithmetischen und geometrischen Mittel durch eine Untersuchung auf Extrema der Funktion

tmo Auf diesen Beitrag antworten »

Kleine Erinnerung, dass hier auch noch was offen steht Wink
Mystic Auf diesen Beitrag antworten »

Ok, ich versuch's dann mal

Zitat:

Aufgabe 25

Man zeige die Ungleichung vom arithmetischen und geometrischen Mittel durch eine Untersuchung auf Extrema der Funktion



Lösung: Zunächst sieht man, dass die verwandte Aufgabe, das Maximum von



zu bestimmen, auf obige führt, wenn man o.B.d.A annimmt und Zähler und Nenner durch dividiert und anschließend



setzt...Setzt man ferner, so erhält man durch Ableiten von



Durch Nullsetzen dieser Ableitungen, sieht man, dass es nur einen stationären Punkt geben kann, dessen Komponenten zunächst alle gleich, nämlich (1+s)/(n+1), und folglich daher alle 1 sind...

Um die Art des stationären Punkts zu bestimmen, berechnen wir auch noch die 2.Ableitungen für diesen Punkt:





Die zugehörige Hessematrix ist aber klar negativ definit... Es sollte genügen, am Fall n=3 darzustellen warum, im allgemeinen Fall funktioniert das genauso... Hier ist die Hessematrix (bis auf den Faktor 1/64) gegeben durch die Matrix



und die zugehörige quadratische Form



ist klarerweise niemals positiv...

Wahrscheinlich sind in obigen Ausführungen noch jede Menge Fehler enthalten, aber ich muss jetzt leider Schluss machen...


Edit: Habe die Definition von s (s. Anmerkung von tmo) und einige kleine Tippfehler noch berichtigt...
tmo Auf diesen Beitrag antworten »

Deine weiteren Rechnungen legen nahe, dass du eher definieren wolltest, oder?

Es fehlt noch ein Argument, dass das Maximum auch wirklich global ist.

Wenn man dieses Argument bringt, so sieht man auch gleich, dass man sich die Hessematrix sparen kann:


Betrachte das Kompaktum .

Für ist für .

Daher nimmt sein globales Maximum für hinreichend große nicht auf dem Rand an. Da es im Inneren nur einen kritischen Punkt gibt, muss das globale Maximum dort angenommen werden. Also gilt:

für alle

Nächste Aufgabe würde ich sagen Freude
Mystic Auf diesen Beitrag antworten »

Ja, in der Definition von s war natürlich der Summand 1 zuviel (das habe ich oben inzwischen ausgebessert)... Dass keine Randmaxima vorliegen hatte ich zwar für mich auf die gleiche Art überlegt, aber dann leider vergessen hinzuschreiben... Was schließlich die Hessematrix betrifft, so hatte mich einfach der Ehrgeiz gepackt, zu zeigen, dass man die selbst für allgemeines n für den einzigen stationären Punkt noch relativ einfach bestimmen kann, obwohl ich ahnte, dass dies nicht der "vorgesehene" Weg ist... Augenzwinkern

Ok, dann auf ein Neues und hoffentlich noch nicht Bekanntes:

Zitat:

Aufgabe 26: 100 Gefangene, welche praktischerweise mit den Nummern 1-100 versehen sind, sehen ihrer Hinrichtung entgegen, sie haben aber noch die Chance der Begnadigung, wenn sie den nachfolgenden Test erfolgreich bestehen. Dabei wird einer nach dem anderen in einen Raum geführt , in dem 100 ungeöffnete Behälter aufgereiht stehen, wobei in jedem ein Zettel mit (Überraschung!) genau einer der Zahlen 1-100 (aber keiner doppelt!) liegt... Jeder öffnet nun maximal 50 Behälter sein Wahl und falls er dabei einen Zettel findet mit der Nummer, welcher seiner eigenen entspricht, ist für ihn selbst der Test erfolgreich verlaufen... Das Perfide an der Sache ist aber, dass für alle 100 Gefangenen der Test erfolgreich sein muss, damit sie begnadigt werden...

Obwohl keinerlei Kommunkation während des eigentlichen Testablaufs möglich ist, haben sich die Gefangenenen aber vorher eine gemeinsame Strategie zurechtgelegt: Jeder geht zuerst zu dem Behälter mit der Nummer, welcher seiner eigenen entspricht... Findet er dort den passenden Zettel, kann er den Test vorzeitig (erfolgreich) abbrechen, ansonsten interpretiert er die Nummer des Zettels als "Zeiger" zum nächsten Behälter usw. Wenn er auf diese Weise nach maximal 50 Versuchen "seinen" Zettel noch nicht gefunden hat, dann war der Test insgesamt für alle 100 Gefangenen erfolglos und sie sehen ihrer Hinrichtung entgegen... Wie groß ist die Wahrscheinlichkeit, dass dies nicht der Fall ist?
HAL 9000 Auf diesen Beitrag antworten »

Es geht im Grunde um die Wahrscheinlichkeit, ob eine 100er-Permutation ausschließlich Zykel der Länge aufweist. Und diese Wahrscheinlichkeit müsste gleich



sein. verwirrt


P.S.: Interessant wäre dieselbe Problemstellung mit einer Maximalzahl kleiner als 50. Denn dann geht es nicht mehr so einfach wie oben von mir gerechnet - denn dort war ja nutzbar, dass eine 100er Permutation maximal einen Zykel von einer Länge größer als 50 haben kann.
Mystic Auf diesen Beitrag antworten »

Ja, das ist natürlich richtig ... Freude

Wobei man hier natürlich auch als gute Approximation nehmen könnte... Vermutlich war das für einen Profi viel zu einfach... Ich habe mich aber nicht getraut, 50 durch eine kleinere Zahl zu ersetzen, denn dann wird es echt haarig... Augenzwinkern

Wie auch immer, du bist dran...
HAL 9000 Auf diesen Beitrag antworten »

Zitat:
Original von Mystic
Ich habe mich aber nicht getraut, 50 durch eine kleinere Zahl zu ersetzen, denn dann wird es echt haarig... Augenzwinkern

Hehe, dazu hatte ich gerade eben noch einen P.S. angebracht. Big Laugh
tmo Auf diesen Beitrag antworten »

Da sieht man mal wieder, dass Mathematik Leben retten kann Big Laugh

Ersetzt man 50 und 100 durch n und 2n, so konvergiert die Wahrscheinlichkeit des Überlebens gegen , während das zufällige Öffnen der Behälter für große n sehr sicher zum Tod führt Teufel
HAL 9000 Auf diesen Beitrag antworten »

Hab gerade keine Lust, mir was auszudenken bzw. auszusuchen. Also wer will, kann mir das diesmal gern abnehmen.
tmo Auf diesen Beitrag antworten »

Ok, dann mal ein Quickie:

Aufgabe 27:

Es sei prim. Man zeige, dass in ganzen Zahlen nur die triviale Lösung (0,0,0) hat.
Mystic Auf diesen Beitrag antworten »

Ich denke, so müsste es gehen:

Lösung zu Aufg. 27:

Gäbe es eine nichtriviale Lösung in ganzen Zahlen, so folgt aus natürlich sofort, dass p|x und daher mit nach Einsetzen und Kürzen durch p auch



Wir haben damit exakt die gleiche Situation, wie am Anfang vorliegen, nur mit vertauschten Rollen der Variablen... Jedenfalls können wir, indem wir so weitermachen, z.B. mit der Betragssummennorm zu immer kleineren nichttrivialen Lösungen gelangen (Stichwort: Unendlicher Abstieg), was natürlich nicht geht...

Edit: Ok, es ist vielleicht doch eine Spur komplizierter, als oben dargestellt... Wenn x nämlich schon 0 war, dann ist natürlich auch , d.h., die Betragssummennorm des Lösungstripels hätte sich dann nicht verändert... Immerhin aber wechseln wir gleich anschließend zu einer anderen Variablen des Lösungsgtripels (oben ist das y), für welche in alanloger Weise gilt... Einsetzen und Kürzen durch p liefert dann auch eine Darstellung ... Das so entstandene Lösungstripel ist aber dann sicher gegenüber (x,y,z) bez. der Betragssummennorm "kleiner"...
tmo Auf diesen Beitrag antworten »

Ja Freude

Alternativ kann man nach einfach weiterfolgern (mod betrachten), dass und (mod betrachten) gilt.

Dann kann man sofort zu einem neuen Lösungstripel übergehen und hat den unendl. Abstieg.
Mystic Auf diesen Beitrag antworten »

Ok, dann würde ich wieder mal was "Algebraisches" vorschlagen:

Zitat:
Aufgabe 28

Wie sehen die paarweise nichtisomorphen Ringe R der Ordnung p², p prim, mit nichtzyklischer additiver Gruppe aus?

Hinweis: Ein Ring muss dabei nicht notwendgerweise ein Einselement haben bzw. kommutativ sein...Am natürlichsten ist vermutlich eine Unterteilung der Ringe nach ihrem (Jacobson-)Radikal...
tmo Auf diesen Beitrag antworten »

Ich poste einfach mal, was ich habe, auch wenn ich nicht ganz zufrieden damit bin.



Klar ist: R wird als additive Gruppe von zwei Elementen erzeugt, z.b. a und b.

Ferner ist die Ringstruktur auf R eindeutig durch und bestimmt, da jedes Element die Form hat.

Zunächst betrachten wir den Fall, dass R Ring mit 1 ist, also o.B.d.A .

Dann ist und die Ringstruktur wird eindeutig durch bestimmt und R ist insbesondere kommutativ.

Auf jeden Fall erfüllt b eine quadratische Gleichung

Ist f irreduzibel (modulo p), so ist .

Ist f reduzibel und separabel, so ist .

Ist f reduzibel und nicht separabel, so ist .


Nun zu Ringen ohne 1, kommutativ:

Wir gehen wie vorgeschlagen über das Jacobsonradikal.

Zunächst zeigen wir, dass es auf jeden Fall ein Ideal der Größe p gibt:
Sei fest und das von a erzeugte Hauptideal sei kein Ideal der Größe p. Dann ist . Also ist die Abb. surjektiv, also bijektiv, da R endlich ist.
Insbesondere gibt es ein eindeutiges mit .
Wenn es nun kein Ideal der Größe p gibt, gilt dies für alle .

Wir müsses nur noch zeigen, dass alle Einsen gleich sind.
Seien dazu bel. und c so gewählt, dass .
Es gilt , also , also .

Damit ist gezeigt, dass R Integritätsbereich ist, also , wenn es kein echtes Ideal gibt.

Nun wieder zurück zum eigentlichen Thema:

Zunächst betrachten wir .
Wegen obiger Argumentation, gibt es dann 2 Hauptideale mit .
Es gilt , sowie und .
Gilt und so ist ein Einselement, Widerspruch!

Dies lässt noch die beiden Fälle sowie .

Beides lässt ich realisieren, erster Fall z.b. durch

,

der zweite durch

.



Nun kommt der Fall , also z.B. .
Wir wollen ausschließen:
Wir wissen , also mit . Dies liefert , also . Bei n=1 wäre b ein Einselement, bei n = 0 wäre ein Ideal der Größe p, jeweils Widerspruch.
Folglich gilt , also ist mit der trivialen Multiplikation ausgestattet, was , also o.b.d.A. liefiert.

Da das Jacobsonradikal keine idempotenten Elemente enthält, muss gelten, was zu , also führt, womit der Ring eindeutig durch festgelegt ist.

Man kann ihn so realisieren:



und


Zu den nicht-kommutativen Ringen: Wenn ich deinen Hinweis nutze, erhalte ich

, also .

Ähnlich wie oben erhält man .

Wegen der Nicht-Kommutativität erhält man also die Möglichkeiten:




Die beiden Möglichkeiten müsste man dann über geeignete p x p-Matrizen realisieren können.


Ich weiß aber nicht, ob das alles so gerechtfertigt ist, weil doch die meisten Eigenschaften des Jacobsonradikals besitmmt kaputt gehen, sobald man keine 1 mehr hat. verwirrt


Insgesamt hätte ich dann 8 verschiedene solche Ringe.
Es wäre schön, wenn du deine Lösung präsentieren könntest. Du kannst dann auch gleich die nächste Aufgabe stellen, wenn du willst, so richtig gelöst habe ich die hier ja nicht.
Mystic Auf diesen Beitrag antworten »

Zitat:
Original von tmo

Ich weiß aber nicht, ob das alles so gerechtfertigt ist, weil doch die meisten Eigenschaften des Jacobsonradikals besitmmt kaputt gehen, sobald man keine 1 mehr hat. verwirrt


Insgesamt hätte ich dann 8 verschiedene solche Ringe.
Es wäre schön, wenn du deine Lösung präsentieren könntest. Du kannst dann auch gleich die nächste Aufgabe stellen, wenn du willst, so richtig gelöst habe ich die hier ja nicht.

Ja, perfekt... Freude

Keine Ahnung, warum du mit deiner Lösung so unzufrieden bist, es stimmt ja alles... Augenzwinkern Was das Jacobsonradikal betrifft, so braucht ja man nur, dass es speziell für endliche Ringe einfach das größte nilpotente Ideal im Ring ist und jedes Nilideal des Rings enthält, und mehr hast du ja auch nicht verwendet...

Ich selber hätte jetzt nur eine leicht andere Systematik gewählt, die aber dafür auch mehr auf Sätzen der Ringtheorie aufbaut... Prinzipiell sei vorausgeschickt, dass jeder Ring der Aufgabenstellung in eindeutiger Weise (die Skalarmultikation entspricht dabei genau der Bildung additiver Potenzen in der Gruppe) eine Algebra über , d.h., insbesondere ein Vektorraum über ist... Wenn ich im Folgenden also von "Basen" rede werde, bezieht sich das immer auf diese Vektorraumstruktur...

1. Fall: J(R)=(0)

Nach den Struktursätzen von Wedderburn-Artin für halbeinfache Ringe ist dann R eine direkte Summe von endlichen vielen vollen Matrizenringen über Schiefkörpern... Aus Schiefkörpern werden natürlich (wieder nach einem bekannten Satz von Wedderburn) im endlichen Fall dann Körper und da wir über Ringe der Ordung p² reden, "verkümmern" die Matrizen zu 1x1-Matrizen, d.h., unsere Ringe sind einfach endliche direkte Summen von Körpern... Das führt also dann mal auf die 2 Fälle bzw. ...

2. Fall: |J(R)|=p

Da der Ring dann nicht nilpotent ist, muss es ein Idempotent geben... Ich wähle, dann meine Basis {a,b} so, dass b ein Idempotent und a ein erzeugendes Element von J(R) ist... Daraus ergibt sich aber sofort



für gewisse ... Nun sind aber die Assoziativiätsbedingungen für die Ringmultiplikation (insbesondere (ab)b= a(bb) bzw. b(ba) =(bb)a ) genau dann erfüllt, wenn gilt... Die 4 resultierenden Ringe, die also dann gegeben sind durch die Bedingungen






sind dann klarerweise nichtisomorph, da der erste kommutativ ohne 1, der zweite und dritte nichtkommutativ und antiisomorph sind, der vierte kommutativ mit 1 ist... Tatsächlich sind das natürlich alles wohlbekannte Ringe, nämlich

, wobei hier den Nullring mit p Elementen bezeichnet


(wie ja auch bereits von dir angegeben)

3.Fall: J(R)=R

Das ist normalerweise der schwierigste Fall, hier aber der leichteste... Denn da R dann nilpotent mit Nilpotenzgrad höchstens 3 sein muss, ist R entweder der Nullring mit p² Elementen oder es gibt ein , sodass ... In diesem Fall wähle ich natürlich {a,b} als Basis und erhalte die Multiplikation, welche gegeben ist durch



Diesen weiteren Ring kann man auch darstellen als , wenn man will...

So, das war jetzt meine Sicht der Dinge, wobei ich für Nachfragen jederzeit offen bin, aber ich bin, wie gesagt, mit deinen Ausführungen sehr einverstanden und daher bist jetzt auch du wieder dran... smile
tmo Auf diesen Beitrag antworten »

Zitat:
Original von Mystic
Was das Jacobsonradikal betrifft, so braucht ja man nur, dass es speziell für endliche Ringe einfach das größte nilpotente Ideal im Ring ist und jedes Nilideal des Rings enthält, und mehr hast du ja auch nicht verwendet...


Ah ok, das erklärt auch, warum du den Fall bei eingeordnet hast.

Ich habe die ganze Zeit an die Definition als der Schnitt der maximalen Ideale (wir ham hier zwar keine 1, welche uns mit dem Zornschen Lemma die Existenz maximaler Ideale garantiert, aber endliche Ringe haben ja trivialerweise max. Ideale) gedacht und daher J(R)=R ausgeschlossen. Ich habe mich da eigentlich auf sehr dünnem Eis bewegt und würde es fast als Zufall einstufen, dass am Ende mit den 8 Ringen alles passt Big Laugh

Nichtdestotrotz kommt dann mal die Aufgabe 29, die man auch durch ein einfaches Ansetzen mit einer Faktorisierung lösen kann (Vielleicht wird der Andrang so ja größer).

Zitat:
Zeige, dass reduzibel über jedem Körper der Charakteristik ist. Ferner bestimme man diejenigen (prim), für die schon in in Linearfaktoren zerfällt.
Mystic Auf diesen Beitrag antworten »

Ja, ist eine sehr nette Aufgabe und ich denke, ich weiß, wie's geht... Aber ich möchte hier nicht vorpreschen, bevor sich nicht andere an der Lösung wenigstens versucht haben... Augenzwinkern
Mystic Auf diesen Beitrag antworten »

So, nachdem ich doch eine gehörige Anstandsfrist habe verstreichen lassen und sich HAL offenbar schmollend zurückgezogen hat, gelüstet es mich doch, diesen ausgesprochenen Appetithappen zu verspeisen... Augenzwinkern

Ich möchte dazusagen, dass ich mich dabei nicht allzu sehr an die Vorgaben von tmo gehalten habe, wenngleich die Grundidee natürlich sehr ähnlich ist, denn ich wollte ja auch zeigen, dass man auch durchaus ohne allzu "schwere Geschütze" aus der Algebra auskommt...Meine Lösung stützt sich denn auch mehr auf Sätze der Zahlentheorie und entspricht auch in etwa meiner ersten Idee, als ich die Aufgabe zum erstenmal sah...

Zitat:
Lösung von Aufgabe 29

Was ich im folgenden verwende ist, dass

- die prime Restklassengruppe zyklisch und g eine Primitivwurzel ist
- die quadratischen Reste genau die Potenzen mit geradem k sind
- in gilt

Sollte davon irgendwas nicht bekannt sein, bin ich gerne bereit, den Beweis dafür nachzubringen...

Da für p=2 natürlich gilt, habe ich folgende Fallunterscheidung vorgenommen:

1. Fall:

Hier hat man dann sofort die Zerlegung



d.h., das Polynom ist reduzibel... Damit es in Linearfaktoren zerfällt, müsste nach obigem (p-1)/4 gerade sein... Ist dies aber erfüllt, so ist dann tatsächlich

mit

eine Zerlegung in Linearfaktoren...

2.Fall:

In diesem Fall ist -1 (=) ein quadratischer Nichtrest da ja (p-1)/2 dann ungerade ist und daher in jedem Fall entweder 2 oder -2 quadratischer Rest

In jedem Fall ist daher eine der beiden Darstellungen



mit der 3.Binomischen Formel weiter zerlegbar d.h., das Polynom ist jedenfalls reduzibel...

Die Zerlegbarkeit in Linearfaktoren ist aber in diesem Fall keinesfalls gegeben, da -1 quadratischer Nichtrest und damit



unlösbar ist...

Insgesamt, können wir sagen, dass die Zerlegbarkeit in Linearfaktoren für p>2 äquivalent damit ist, dass (p-1)/8 ganz ist, wie sich dies aus dem ersten Fall ergibt...


Edit: Wie man auch sieht, hat gegenüber dem, was HAL hier schon beigetragen hat, wirklich nur mehr sehr wenig gefehlt...

Edit2: Sorry, dass die Erstversion noch unglaublich viele Fehler enthielt, ich hoffe aber, inzwischen stimmt alles... Augenzwinkern
Neue Frage »
Antworten »



Verwandte Themen

Die Beliebtesten »
Die Größten »
Die Neuesten »